東京大学大学院 情報理工学研究科 数学 2016年度 第1問 解答


(1)

明らかに,  A = \begin{bmatrix}1 & 1 & 1\\1 & 0 & 0 \\ 0 & 1 & 0 \end{bmatrix} である.

(2)

明らかに, A のランクは  \text{rank} \begin{bmatrix}1 & 1 & 1\\1 & 0 & 0 \\ 0 & 1 & 0 \end{bmatrix} = 3 である.
明らかに, A特性方程式 t^3 - t^2 - t - 1 = 0 である.

(3)

固有値  \lambda_i,  (i = 1, 2, 3)に対応する固有ベクトルを自明な方法で求めると,  \begin{bmatrix} \lambda_i^2 \\ \lambda_i \\ 1 \end{bmatrix} の定数倍で表せる.

(4)

写像 f:\mathbb{R} \rightarrow \mathbb{R} f(t) = t^3 - t^2 - t - 1 = 0 と定めると,  t多項式より, 連続だから, その制限写像  f|_{[0, 2]}も連続である.
 f(1) = -2 < 0,  f(2) = 1 > 0 だから, 中間値の定理より,  f(c) = 0 を満たす  c \in (1, 2) が存在する.
また, 高校数学より増減表を書くと, グラフ  f x軸の交点は  t = c のみだとわかる.
よって, 主張を得る.

(5)

  \begin{bmatrix} T_{n+3} \\ T_{n+2} \\ T_{n+1} \end{bmatrix} = A \begin{bmatrix} T_{n+2} \\ T_{n+1} \\ T_n \end{bmatrix} =  A^{n+1} \begin{bmatrix} T_2 \\ T_1\\ T_0   \end{bmatrix} = A^{n+1} \begin{bmatrix} 1 \\ 0 \\ 0 \end{bmatrix} である.
 (1) (3) より,  A は対角化可能で,  P := \begin{bmatrix} \lambda_1^2 & \lambda_2^2 & \lambda_3^2 \\ \lambda_1 & \lambda_2 & \lambda_3 \\ 1 & 1 & 1\end{bmatrix} とおくと,
\begin{align}
P^{-1} A P &= \begin{bmatrix} \lambda_1 & 0 & 0 \\ 0 & \lambda_2 & 0 \\ 0 & 0 & \lambda_3 \end{bmatrix} \\
A^{n+1} &= P \begin{bmatrix} \lambda_1^{n+1} & 0 & 0 \\ 0 & \lambda_2^{n+1} & 0 \\ 0 & 0 & \lambda_3^{n+1} \end{bmatrix} P^{-1}
\end{align}
 P  \begin{bmatrix} \lambda_1^{n+1} & 0 & 0 \\ 0 & \lambda_2^{n+1} & 0 \\ 0 & 0 & \lambda_3^{n+1} \end{bmatrix} の第 1 行の行ベクトルは \begin{bmatrix} \lambda_1^{n+3} &  \lambda_2^{n+3} & \lambda_3^{n+3} \end{bmatrix} でだから,
  P \begin{bmatrix} \lambda_1^{n+1} & 0 & 0 \\ 0 & \lambda_2^{n+1} & 0 \\ 0 & 0 & \lambda_3^{n+1} \end{bmatrix} P^{-1} (1, 1) 成分は, ある複素定数  c_1, c_2, c_3 を用いて,  c_1 \lambda_1^{n+3} + c_2 \lambda_2^{n+3} + c_3 \lambda_3^{n+3} と表せる.
よって,  T_{n+3} = c_1 \lambda_1^{n+3} + c_2 \lambda_2^{n+3} + c_3 \lambda_3^{n+3}, すなわち,  T_{n} = c_1 \lambda_1^{n} + c_2 \lambda_2^{n} + c_3  \lambda_3^{n}である.

(6)

 t^3 - t^2 - t - 1 = 0 において解と係数の関係より,  \lambda_1 \lambda_2 \lambda_3 = 1 が成立.
 (4) より,  |\frac{\lambda_2}{\lambda_1}||\frac{\lambda_3}{\lambda_1}| = |\frac{1}{\lambda_1}| < 1 だから,   |\frac{\lambda_2}{\lambda_1}| <1 または |\frac{\lambda_3}{\lambda_1}| <1 である.
 \lambda_2 \lambda_3複素共役より,   |\frac{\lambda_2}{\lambda_1}| <1 かつ |\frac{\lambda_3}{\lambda_1}| <1 である.
よって,  \lim_{n \rightarrow \infty} \left(\frac{\lambda_2}{\lambda_1}\right)^n = 0,  \lim_{n \rightarrow \infty} \left(\frac{\lambda_3}{\lambda_1}\right)^n = 0 である.
したがって,
\begin{align}
\lim_{n \rightarrow \infty} \frac{T_{n+1}}{T_n} &= \lim_{n \rightarrow \infty} \frac{c_1 \lambda_1^{n+1} + c_2 \lambda_2^{n+1} + c_3 \lambda_3^{n+1}}{c_1 \lambda_1^{n} + c_2 \lambda_2^{n} + c_3 \lambda_3^{n}} \\
&= \lim_{n \rightarrow \infty} \frac{c_1 \lambda_1 + c_2 \lambda_2 (\frac{\lambda_2}{\lambda_1})^{n} + c_3 \lambda_3 (\frac{\lambda_3}{\lambda_1})^{n}}{c_1 + c_2 (\frac{\lambda_2}{\lambda_1})^{n} + c_3 (\frac{\lambda_3}{\lambda_1})^{n}} \\
& = \lambda_1
\end{align}